Legislator: The recently passed highway bill is clearly very unpopular with voters. After all, polls predict that the...

tomgbean on December 31, 2019

C

Why is C wrong?

Reply
Create a free account to read and take part in forum discussions.

Already have an account? log in

AndreaK on December 31, 2019

Hi @tomgbean,

This is a flaw question. It’s starts out with a big, presumptuous conclusion.

Conclusion: The recently passed highway bill is clearly very unpopular with voters.

So, legislator, that’s a bold statement you have there. It’s clearly very unpopular with voters...so what’s your evidence? Did you poll every single voter out there about the bill? What makes you so sure that it’s clearly unpopular with them?

That’s the kind of critical thinking we should be doing on the LSAT. Remember, be careful taking anything at face value—the benefit of the doubt is for your friends, not your enemies. And the LSAT isn’t on your team.

So, the legislator’s evidence to support this lofty conclusion is...?

The polls predict that the majority party (that supports the bill’s passage) will lose seats in the upcoming election.

Okay, that’s all fine and dandy but...what does that tell us about the favorability of the actual bill? Nothing, right? Think of it this way. What if both parties supported the bill? We aren’t told the fact that one party supports the bill is the reason that it won’t be getting votes.

If you look at A, that gets to this idea in a roundabout way. Admittedly, it might take a little deconstructing to see.

A) Gives no reason to think that the election outcome would be different if the majority party had not supported the bill.

So, remember the assumption being made here is that because the party that supports the bill will be losing seats in the electorate, the public doesn’t support the bill. But remember, we don’t know if it’s actually that bill that’s causing them to lose the votes. So, if the legislators’ argument did give us reason to think that the majority party would not be losing seats if it had not supported the bill, we would know the bill was likely at play here and the argument would make more sense. That’s what we’re missing to make this argument work. As it stands, that assumption it’s relying on is a big place of vulnerability.

C is describing a circular reasoning flaw. Those aren’t very common right answers. Here, there’s no claim that assumes the bill is unpopular. The author is concluding that the bill is unpopular, but there’s not a direct statement in the premises that says so.

Hope this helps!